GMAT Critical Reasoning Discussions

The GMAT and Related Discussions section is very fragmented in terms of the information available at one’s disposal. There are one of reply threads and several other redundant threads that have masked the more important ones. This is an atte…

The GMAT and Related Discussions section is very fragmented in terms of the information available at one's disposal. There are one of reply threads and several other redundant threads that have masked the more important ones.

This is an attempt to make the best of explanations, reasonings, questions etc etc easily available to the junta here. We would be having separate threads for topics such as

Problem Solving (the existing one is renamed)

Data Sufficieny
Sentence Correction
Critical Reasoning

Reading Comprehension.

Please ensure that Q & As are carried out in the repsective threads.

Thread is made sticky

PS:-This post would be edited by the MODS in order to give links to various CRs that would be discussed here. We will try to organise the links for the questions discussed here.
Other relevant posts in the 'one of reply' threads would be moved here

Someone please help me with this question..





A researcher discovered that people who have low levels of immune system activity tend to score much lower on tests of mental health than do people with normal or high immune system activityThe researcher concluded from this experiment that the immune system protects against mental illness as well as against physical disease
The researcher's conclusion depends on which of the following assumptions?
(A) High immunesystem activity protects against mental illness better than normal immune, system activity does
(B) Mental illness is similar to physical disease in its effects on body systems.
(C) People with high immune-system activity cannot develop mental illness
(D) Mental illness does not cause people's immune-system activity to decrease.
(E) Psychological treatment of mental illness is not as effective as is medical treatment.

Someone please help me with this question..

A researcher discovered that people who have low levels of immune system activity tend to score much lower on tests of mental health than do people with normal or high immune system activityThe researcher concluded from this experiment that the immune system protects against mental illness as well as against physical disease
The researcher's conclusion depends on which of the following assumptions?
(A) High immunesystem activity protects against mental illness better than normal immune, system activity does
(B) Mental illness is similar to physical disease in its effects on body systems.
(C) People with high immune-system activity cannot develop mental illness
(D) Mental illness does not cause people's immune-system activity to decrease.
(E) Psychological treatment of mental illness is not as effective as is medical treatment.


I think its B
Because the researcher concludes that the immune system protects against both mental and physical disease.....wheras scores are obtained only of mental tests.....so the assumption is that mental and physical diseases are similar in nature - a fact demonstrated by option B.

Is that the right answer ?...
I think its B
Because the researcher concludes that the immune system protects against both mental and physical disease.....wheras scores are obtained only of mental tests.....so the assumption is that mental and physical diseases are similar in nature - a fact demonstrated by option B.

Is that the right answer ?...


No :(
The answer is D... Someone please explain if u can..
No :(
The answer is D... Someone please explain if u can..


Well someone plz do explain this..
By the way...wats the source of this question ?..






.
Well someone plz do explain this..
By the way...wats the source of this question ?..






.


OG11 verbal review Q no 7


A researcher discovered that people who have low levels of immune system activity tend to score much lower on tests of mental health than do people with normal or high immune system activityThe researcher concluded from this experiment that the immune system protects against mental illness as well as against physical disease
The researcher's conclusion depends on which of the following assumptions?
(A) High immunesystem activity protects against mental illness better than normal immune, system activity does
(B) Mental illness is similar to physical disease in its effects on body systems.
(C) People with high immune-system activity cannot develop mental illness
(D) Mental illness does not cause people's immune-system activity to decrease.
(E) Psychological treatment of mental illness is not as effective as is medical treatment.



The conclusion made by the researcher is that

"the immune system protects against mental illness as well as against physical disease"

because people with low levels of immune system activity scored much lower on tests of mental health than people with normal or high immune system activity

Suppose the low levels of immune system were due to mental illness, in other words mental illness affected the immune system itself then the scoring on the tests of mental health will come under doubt and eventually the conclusion will fall apart.

Thus the assumption researcher makes is that mental illness does not cause people's immune-system activity to decrease

Option D

One question to Govi da Mod are you going to shift all problem from http://www.pagalguy.com/discussions/gmat-problem-solving-discussions-25019823
to all relevant threads?

you can delete this post later

One question to Govi da Mod are you going to shift all problem from http://www.pagalguy.com/discussions/gmat-problem-solving-discussions-25019823
to all relevant threads?

you can delete this post later


Yes we intend to that since many SCs CRs questions have been discussed in that thread and some of them have very good explanations too. But all this would call for time not sure how soon any of the mods would be able to spare that much time but we will try and complete the task in the coming few weeks.

~S

Hello,
I am posting one of the classic critical reasoning problem:
try solving this and know/learn the explanation of the same well.

once you understand such a problem I am sure most of your CR fear is out.

try this:


If A, then B.
If B, then C.
If C, then D.


If all of the statements above are true, which of the following must also be true?


(A) If D, then A.
(B) If not B, then not C.
(C) If not D, then not A.
(D) If D, then E.
(E) If not A, then not D.

Hello,
I am posting one of the classic critical reasoning problem:
try solving this and know/learn the explanation of the same well.

once you understand such a problem I am sure most of your CR fear is out.

try this:


I would pick C - If not D then not A
reasoning - logic of If X then Y If not Y then not X
Hello,
I am posting one of the classic critical reasoning problem:
try solving this and know/learn the explanation of the same well.

once you understand such a problem I am sure most of your CR fear is out.

try this:



Ill also pick option C

Coz,

If X ==> Y then, not y ==> not X

only c gives that option. hence C

well, yes that is the correct answer and correct approach.

Which of the following best completes the passage below?


At a recent conference on environmental threats to the North Seamost participating countries favored uniform controls on the quality of effluentswhether or not specific environmental damage could be attributed to a particular source of effluentWhat must, of course, be shown, in order to avoid excessively restrictive controlsis that
(A) any uniform controls that are adopted are likely to be implemented without delay
(B) any substance to be made subject to controls can actually cause environmental damage
(C) the countries favoring uniform controls are those generating the largest quantities of effluents
(D) all of any given pollutant that is to be controlled actually reaches the North Sea at present
(E) environmental damage already inflicted on the North Sea is reversible

Which of the following best completes the passage below?


At a recent conference on environmental threats to the North Seamost participating countries favored uniform controls on the quality of effluentswhether or not specific environmental damage could be attributed to a particular source of effluentWhat must, of course, be shown, in order to avoid excessively restrictive controlsis that
(A) any uniform controls that are adopted are likely to be implemented without delay
(B) any substance to be made subject to controls can actually cause environmental damage
(C) the countries favoring uniform controls are those generating the largest quantities of effluents
(D) all of any given pollutant that is to be controlled actually reaches the North Sea at present
(E) environmental damage already inflicted on the North Sea is reversible




I would go with B. It is the logically correct answer. The others, i feel, go out of the scope.
Whats the OA??
I would go with B. It is the logically correct answer. The others, i feel, go out of the scope.
Whats the OA??


Ya the answer is B. But can anyone explain why D is out of scope?
Which of the following best completes the passage below?


At a recent conference on environmental threats to the North Seamost participating countries favored uniform controls on the quality of effluentswhether or not specific environmental damage could be attributed to a particular source of effluentWhat must, of course, be shown, in order to avoid excessively restrictive controlsis that
(A) any uniform controls that are adopted are likely to be implemented without delay
(B) any substance to be made subject to controls can actually cause environmental damage
(C) the countries favoring uniform controls are those generating the largest quantities of effluents
(D) all of any given pollutant that is to be controlled actually reaches the North Sea at present
(E) environmental damage already inflicted on the North Sea is reversible




Point D simply states that for the pollutant which is to be controlled, all of it is reaching the North Sea ( not a small portion)....Thats why its out of scope of the question.

Can someone pls help me with following question?


Partly because of bad weather, but also partly because some major pepper growers have switched to high-priced cocoa, world production of pepper has been running well below worldwide sales for three years. Pepper is consequently in relatively short supply. The price of pepper has soared in response: it now equals that of cocoa.

Which of the following can be inferred from the passage?
(A) Pepper is a profitable crop only if it is grown on a large scale.
(B) World consumption of pepper has been unusually high for three years.
(C) World production of pepper will return to previous levels once normal weather returns.
(D) Surplus stocks of pepper have been reduced in the past three years.
(E) The profits that the growers of pepper have made in the past three years have been unprecedented.




Can someone pls help me with following question?


Partly because of bad weather, but also partly because some major pepper growers have switched to high-priced cocoa, world production of pepper has been running well below worldwide sales for three years. Pepper is consequently in relatively short supply. The price of pepper has soared in response: it now equals that of cocoa.

Which of the following can be inferred from the passage?
(A) Pepper is a profitable crop only if it is grown on a large scale.
(B) World consumption of pepper has been unusually high for three years.
(C) World production of pepper will return to previous levels once normal weather returns.
(D) Surplus stocks of pepper have been reduced in the past three years.
(E) The profits that the growers of pepper have made in the past three years have been unprecedented.






E??
Other option deal with effects of increase in production of pepper which can not be directly inferred from the passage.